Đến nội dung

viet9a14124869 nội dung

Có 855 mục bởi viet9a14124869 (Tìm giới hạn từ 29-04-2020)



Sắp theo                Sắp xếp  

#672267 Topic về phương trình và hệ phương trình

Đã gửi bởi viet9a14124869 on 21-02-2017 - 13:15 trong Phương trình - hệ phương trình - bất phương trình

Xin đóng góp hai bài khá dễ sau 

Bài 555 : Giải hệ phương trình :

   $\left\{\begin{matrix} xy+x+y=7 & & \\ x^3+y^3+3(x^2+y^2)+3(x+y)=70 & & \end{matrix}\right.$

 

Bài 556:Giải hệ phương trình :

  $\left\{\begin{matrix} (x-4)\sqrt{y-3}+(y-1)\sqrt{x+2}=7\sqrt{6} & & \\ 12x\sqrt{y-4}+4y\sqrt{2(x-2)}=5xy & & \end{matrix}\right.$




#671327 Topic về phương trình và hệ phương trình

Đã gửi bởi viet9a14124869 on 12-02-2017 - 20:53 trong Phương trình - hệ phương trình - bất phương trình

Bài 554: $\left\{\begin{matrix} &\sqrt{5-x^{2}}+\sqrt{5-\dfrac{1}{x^{2}}}=3+y^{2} \\ &x+\dfrac{1}{x}=2(3-2y) \end{matrix}\right.$

Theo bdt Cô-si ta có $12+4y^2=2\sqrt{4(5-x^2)}+2\sqrt{4(5-\frac{1}{x^2})}\leq (4+5-x^2)+(4+5-\frac{1}{x^2})\leq 18-\frac{1}{2}(x+\frac{1}{x})^2=18-2(3-2y)^2$

$\Leftrightarrow 12(y-1)^2\leq 0\Rightarrow y=1\Rightarrow x=1$




#676656 Topic về Bất đẳng thức, cực trị THCS

Đã gửi bởi viet9a14124869 on 08-04-2017 - 21:12 trong Bất đẳng thức và cực trị

Cho a,b,c>0 và a^2+b^2+c^2=1.CM $\frac{bc}{a^2+1}+\frac{ca}{b^2+1}+\frac{ab}{c^2+1}\leqslant \frac{3}{4}$

Ta có $\frac{b^2}{a^2+b^2}+\frac{c^2}{c^2+a^2}\geq \frac{(b+c)^2}{2a^2+b^2+c^2}\geq \frac{4bc}{2a^2+b^2+c^2}=\frac{4bc}{a^2+1}$

Làm tương tự rồi cộng 3 vế lại ta có đpcm !

Dấu = xảy ra khi $a=b=c=\frac{1}{\sqrt{3}}$




#672777 Topic về Bất đẳng thức, cực trị THCS

Đã gửi bởi viet9a14124869 on 25-02-2017 - 22:21 trong Bất đẳng thức và cực trị

Cho a,b,c>0 thỏa $a+b+c\geq 3$.Tìm giá trị nhỏ nhất của biểu thức S=$\frac{a}{\sqrt{b}}+\frac{b}{\sqrt{c}}+\frac{c}{\sqrt{a}}$

Cho x,y,z>0 thỏa mãn x+y+z=1. Tìm giá trị lớn nhất của biểu thức: P=$\frac{x}{x+1}+\frac{y}{y+1}+\frac{z}{z+1}$

1, Ta có $\frac{a}{\sqrt{b}}+\frac{b}{\sqrt{c}}+\frac{c}{\sqrt{a}}=\frac{a^2}{a\sqrt{b}}+\frac{b^2}{b\sqrt{c}}+\frac{c^2}{c\sqrt{a}}\geq \frac{(a+b+c)^2}{a\sqrt{b}+b\sqrt{c}+c\sqrt{a}}\geq \frac{(a+b+c)^2}{\sqrt{(a+b+c)(ab+bc+ca)}}=\frac{9}{\sqrt{3(ab+bc+ca)}}\geq 3\Leftrightarrow a=b=c=1$

2, Bạn thay x+y+z=1 vào mẫu mỗi phân số rồi dùng cô-si là xong




#675998 Topic về Bất đẳng thức, cực trị THCS

Đã gửi bởi viet9a14124869 on 02-04-2017 - 15:55 trong Bất đẳng thức và cực trị

Cho a,b,c là các số thực dương thỏa mãn abc=1.Chứng minh:

$\frac{a}{(a+1)(b+1)}+\frac{b}{(b+1)(c+1)}+\frac{c}{(c+1)(a+1)}\geq \frac{3}{4}$

Cho ba số dương a,b,c thỏa $a+b+c\leq k$ thì $(1+\frac{1}{a})(1+\frac{1}{b})(1+\frac{1}{c})\geq (1+\frac{3}{k})^3$

Câu đầu đồng quy biểu thức ,,,ta cần chứng minh $ab+bc+ca+a+b+c\geq 6$ đúng theo AM-GM

Câu sau ta dùng holder với AM-GM ta có $\prod (\frac{1}{2}+\frac{1}{2}+\frac{1}{a})\geq (1+\frac{1}{\sqrt[3]{abc}})^3\geq (1+\frac{3}{a+b+c})^3\geq (1+\frac{3}{k})^3$




#674047 Topic về Bất đẳng thức, cực trị THCS

Đã gửi bởi viet9a14124869 on 12-03-2017 - 13:18 trong Bất đẳng thức và cực trị

Cho x,y thỏa mãn

$\left ( x^{2}+y^{2}+1 \right )^{2}+4x^{2}y^{2}-x^{2}-y^{2}=0$

Tìm min, max P=$x^{2}+y^{2}$

Đề sai bạn ạ ,,,khi tách hết ra ta được $x^4+6x^2y^2+y^4+x^2+y^2+1=0$ vô lí




#677410 Topic về Bất đẳng thức, cực trị THCS

Đã gửi bởi viet9a14124869 on 14-04-2017 - 20:35 trong Bất đẳng thức và cực trị

1. Cho 3 số  dưeơng a,b,c. CMR: $(\frac{a}{b}+\frac{b}{c}+\frac{c}{a})^{2}\geq(a+b+c)(\frac{1}{a}+\frac{1}{b}+\frac{1}{c})$

2. Cho các só thực x,y, z thỏa mãn điều kiện: x2 +y+z2=1. Tìm GTLN của A=xy+yz+2xz

3. Cho các số thực a,b,c thuộc đoạn [-2;5] tm: a+2b+3c$\leq$2. Tìm GTLN: a2+2b2+3c2

4. Cho a,b,c>0 tm: $\frac{1}{a}+\frac{1}{b}+\frac{1}{c}$=1. CMR:$\frac{a^{2}}{a+bc}+\frac{b^{2}}{b+ca}+\frac{c^{2}}{c+ab}\geq \frac{a+b+c}{4}$

Bài 2 , Ta có $\left\{\begin{matrix} xy\leq \frac{\sqrt{3}-1}{2}x^2+\frac{1}{2\sqrt{3}-2}y^2 & & \\ yz\leq \frac{\sqrt{3}-1}{2}z^2+\frac{1}{2\sqrt{3}-2}y^2 & & \\ 2xz\leq x^2+z^2 & & \end{matrix}\right.\Rightarrow xy+yz+2zx\leq \frac{\sqrt{3}+1}{2}(x^2+y^2+z^2)=\frac{\sqrt{3}+1}{2}\Leftrightarrow x=z=\frac{\sqrt{3}+1}{2}y\Leftrightarrow .........$

Bài 3 , $-2\leq a\leq 5\Leftrightarrow a^2\leq 3a+10 \Rightarrow ..... \Rightarrow a^2+2b^2+3c^2\leq (3a+10)+2(3b+10)+3(3c+10)=3(a+2b+3c)+60\leq  66\Leftrightarrow a=-2,b=5,c=-2$

Bài 4 , Chắc là dùng $\frac{a^2}{a+bc}=a-\frac{abc}{a+bc}$ rồi dùng AM-GM ở mẫu ^_^




#677033 Topic về Bất đẳng thức, cực trị THCS

Đã gửi bởi viet9a14124869 on 11-04-2017 - 06:06 trong Bất đẳng thức và cực trị

cho các số thực $a,b,c$. Chứng minh :

$$\frac{ab}{a+b-c}+\frac{bc}{b+c-a}+\frac{ca}{c+a-b}\ge a+b+c$$

 

Đề ra cho là sai $a,b,c\ge 0$ mới đúng

Mình nghĩ nên có thêm điều kiện a,b,c là 3 cạnh tam giác sẽ đúng hơn ,,bởi nếu a=1,b=2,c=4 thì bất đẳng thức sai




#677413 Topic về Bất đẳng thức, cực trị THCS

Đã gửi bởi viet9a14124869 on 14-04-2017 - 20:45 trong Bất đẳng thức và cực trị

Nếu sử dụng $AM-GM$ dưới mẫu sẽ không thỏa mãn dấu bằng vì $a=b=c=3$ mà.

Bạn tách ra $a+bc=a+\frac{bc}{3}+\frac{bc}{3}+\frac{bc}{3}$ rồi dùng AM-GM  là OK




#671120 Topic tổng hợp một số bất đẳng thức trong kì thi MO các nước

Đã gửi bởi viet9a14124869 on 11-02-2017 - 19:01 trong Bất đẳng thức - Cực trị

hình như có người gửi rồi hay sao ý ^-^

Bài 168(Hungary MO): Cho a,b,c>0. CMR nếu a+b+c= 2(ab+ca+ab). Tìm Min biểu thức sau:

P=$\sum \frac{a^3+b^2}{a^2+b^2}$

Ta chứng minh một bổ đề sau

$\frac{a^3+b^3}{a^2+b^2}\geq \frac{a+b}{2}\Leftrightarrow 2(a^3+b^3)\geq (a^2+b^2)(a+b)\Leftrightarrow (a-b)^2(a+b)\geq 0$ (luôn đúng)

Mặt khác $a+b+c=2ab+2bc+2ca\leq \frac{2}{3}(a+b+c)^2\Rightarrow a+b+c\geq \frac{3}{2}$

Do đó $\sum \frac{a^3+b^3}{a^2+b^2}\geq \sum \frac{a+b}{2}=a+b+c\geq \frac{3}{2}\Leftrightarrow a=b=c=\frac{1}{2}$




#678199 $\boxed{\text{Chuyên Đề}}$ Phương trình vô tỉ - Hệ phương...

Đã gửi bởi viet9a14124869 on 21-04-2017 - 12:57 trong Phương trình, hệ phương trình và bất phương trình

Bác nào làm hộ em với  :D

Giải phương trình $\sqrt{4x^2+5x+1}-2\sqrt{x^2-x+1}=9x-3$ 

 

P/s : Em mới có nick trên diễn đàn ,,,,mong sau này được mọi người chiếu cố cho !  :D

Liên hợp : 

Ta thấy $9x-3=\sqrt{4x^2+5x+1}-2\sqrt{x^2-x+1}=\frac{9x-3}{\sqrt{4x^2+5x+1}+2\sqrt{x^2-x+1}}\Leftrightarrow x=\frac{1}{3}$ 

Hoặc $\sqrt{4x^2+5x+1}+2\sqrt{x^2-x+1}=1$ 

Mặt khác $\sqrt{4x^2+5x+1}+2\sqrt{x^2-x+1}\geq 2\sqrt{x^2-x+1}> 1$ (vô lí ) 

Vậy x=$\frac{1}{3}$ là nghiệm duy nhất của phương trình   :)) 




#677626 $\boxed{\text{Chuyên Đề}}$ Phương trình vô tỉ - Hệ phương...

Đã gửi bởi viet9a14124869 on 16-04-2017 - 20:48 trong Phương trình, hệ phương trình và bất phương trình

giải hộ mk bài này với:

$\frac{x^{2}}{25}+\frac{9}{x^{2}}-\frac{11x}{25}+\frac{33}{5x}=0$

Đặt ẩn phụ là ra ngay thôi !

Cho $\frac{x}{5}-\frac{3}{x}=a\Rightarrow$ phương trình đã cho tương đương  0 = $a^2-\frac{11}{5}a+\frac{6}{5}\Rightarrow a\in \left \{ 1,\frac{6}{5} \right \}$

Đến đây xét từng trường hợp nhé

Ra các nghiệm là $x\in \left \{ \frac{5\pm \sqrt{85}}{2},3\pm 2\sqrt{6} \right \}$ :icon6: 




#675844 $\boxed{\text{Chuyên Đề}}$ Phương trình vô tỉ - Hệ phương...

Đã gửi bởi viet9a14124869 on 31-03-2017 - 21:20 trong Phương trình, hệ phương trình và bất phương trình

$(1)\Leftrightarrow (x-2y)(x-y-2)=0$

MỌi NGƯỜI LÀM HỘ MÌNH CÂU NÀY NỮA NHÉ !!!!!!!!!!!!

Giải hệ phương trình:

$\Rightarrow \left\{\begin{matrix}x^{2}+2y^{2}-3xy-2x+4y=0 & & \\ (x^{2}-5)^{2}=2x-2y+5 & & \end{matrix}\right.$

Bạn xét 2TH rồi thế tìm nghiệm nhé !!!




#674891 $\boxed{\text{Chuyên Đề}}$ Phương trình vô tỉ - Hệ phương...

Đã gửi bởi viet9a14124869 on 20-03-2017 - 21:09 trong Phương trình, hệ phương trình và bất phương trình

$\Leftrightarrow 2x^2-8=\sqrt{\frac{x+7}{x+1}}-\sqrt{2x-1}=\frac{8-2x^2}{\sqrt{(x+1)(x+7)}+\sqrt{(x+1)(2x-1)}}\Rightarrow 2x^2=8\Rightarrow x=2$ theo ĐK của x

Helpppppppppppppppppppppppppp !!!!!!!!!!!!!!!!

Giải phương trình

$\sqrt{\frac{x+7}{x+1}}+8=2x^{2}+\sqrt{2x-1}$




#672578 $\boxed{\text{Chuyên Đề}}$ Phương trình vô tỉ - Hệ phương...

Đã gửi bởi viet9a14124869 on 24-02-2017 - 13:08 trong Phương trình, hệ phương trình và bất phương trình

Các bạn suy nghĩ bài này xem:

giải hệ phương trình:

     5x2+ 2y2+ z=2

     xy+ yz+ xz =1

tách như sau $5x^2+2y^2+z^2=(3x^2+\frac{z^2}{3})+(2x^2+\frac{y^2}{2})+(\frac{2z^2}{3}+\frac{3y^2}{2})\geq 2xy+2yz+2zx=2$ nên ta tìm được x,y,z theo dấu = 




#671787 Topic phương trình, hệ phương trình vô tỉ

Đã gửi bởi viet9a14124869 on 16-02-2017 - 13:35 trong Phương trình, hệ phương trình và bất phương trình

máy mình cũng bị tương tự

giải phương trình 4{x-\sqrt(5-x)}.{\sqrt(5-x) +3}=(x+3)^2

 

bạn nào sửa hộ mih đk máy mih gặp sự cố

Sử dụng bdt cauchy cho 2 số ta có 4ab<=(a+b)^2 ....

Do đó ta cm được VT<=VP 




#671623 Topic phương trình, hệ phương trình vô tỉ

Đã gửi bởi viet9a14124869 on 14-02-2017 - 19:46 trong Phương trình, hệ phương trình và bất phương trình

\[ Bài 2 : \sqrt {4{x^2} + 5x + 1}  + 3 = 2\sqrt {{x^2} - x + 1}  + 9x\]

Ta có $\Leftrightarrow \sqrt{4x^2+5x+1}-\sqrt{4x^2-4x+4}=9x-3\Leftrightarrow \frac{9x-3}{\sqrt{4x^2+5x+1}+\sqrt{4x^2-4x+4}}=9x-3$

Xét $x\neq \frac{1}{3}\Rightarrow 1=\sqrt{4x^2+5x+1}+\sqrt{4x^2-4x+4}\geq \sqrt{3}> 1$ loại

Do đó $x=\frac{1}{3}$




#671625 Topic phương trình, hệ phương trình vô tỉ

Đã gửi bởi viet9a14124869 on 14-02-2017 - 19:51 trong Phương trình, hệ phương trình và bất phương trình

\[ Bài 3 : {x^2} = \sqrt {{x^3} - {x^2}}  + \sqrt {{x^2} - x} \]

Xét thấy x=0 là nghiệm của bài toán

Nếu $x\neq 0\Rightarrow x\geq 1\Rightarrow 2x^2=2\sqrt{x^2(x-1)}+2\sqrt{(x^2-x).1}\leq (x^2+x-1)+(x^2-x+1)=2x^2\Leftrightarrow x^2=x-1=1-x\Leftrightarrow 1=x=0$ loại

Vậy x=0




#671833 Topic phương trình, hệ phương trình vô tỉ

Đã gửi bởi viet9a14124869 on 16-02-2017 - 21:23 trong Phương trình, hệ phương trình và bất phương trình

\[ Bài 1 : {x^3} + 2\sqrt {{{(3x - 2)}^3}}  = 3x(3x - 2)\]

 Đặt $\sqrt{3x-2}=a\Rightarrow x^3+b^3+b^3=3xb^2\Leftrightarrow x+2b=0$ hoặc $x^2-2bx+3b^2=0$ loại

Vậy ta sẽ tìm được x do x+2b=0




#671851 Topic phương trình, hệ phương trình vô tỉ

Đã gửi bởi viet9a14124869 on 17-02-2017 - 06:11 trong Phương trình, hệ phương trình và bất phương trình

Sai ! thử thay số vào đi bạn 

Nếu thay số mà không  ra thì chắc là vô nghiệm




#707245 [TOPIC] ÔN THI BẤT ĐẲNG THỨC $\boxed{\text{THPT CHUYÊN}}$...

Đã gửi bởi viet9a14124869 on 29-04-2018 - 08:38 trong Tài liệu - Đề thi

Bài toán số 108 : Cho a,b,c là các số thực dương thỏa mãn a+b+c=1 . Với q=ab+bc+ca , chứng minh rằng : 

$$\frac{a}{(1-a)^2}+\frac{b}{(1-b)^2}+\frac{c}{(1-c)^2}\geq \frac{1}{2q(1-q)}$$

                                                                                                                                                              ____ Nguyễn Đức Việt ____

 

P/S : Yếu quá :P ....




#707283 [TOPIC] ÔN THI BẤT ĐẲNG THỨC $\boxed{\text{THPT CHUYÊN}}$...

Đã gửi bởi viet9a14124869 on 29-04-2018 - 12:24 trong Tài liệu - Đề thi

Bạn yeutoan89 sửa lại bài đi nhé, gõ tex có vấn đề , với lại bạn quên chưa đánh số thứ tự bài kìa 

 

Bài toán số 110 : Giả sử rằng a,b,c là các số thực dương thỏa mãn a+b+c=1 . Chứng minh rằng : 

$$\sqrt[3]{\frac{1}{a}-4b}+\sqrt[3]{\frac{1}{b}-4c}+\sqrt[3]{\frac{1}{c}-4a}\leq \sqrt[3]{\frac{5}{3}} .(\frac{1}{15abc}+\frac{6}{5})$$ 

 

P/S : Sửa đề rồi nha ai vô chém thử đi .....

                                                                                                                                               ____ Nguyễn Đức Việt____




#707211 [TOPIC] ÔN THI BẤT ĐẲNG THỨC $\boxed{\text{THPT CHUYÊN}}$...

Đã gửi bởi viet9a14124869 on 28-04-2018 - 21:21 trong Tài liệu - Đề thi

105. Cho $a\,,b\,\,> 0$. Chứng minh rằng:

 

$\frac{1}{2 + a + b} + \frac{a}{2a + b + 1} + \frac{b}{2b + a + 1} \,\, \leqq \frac{3}{4}$

Lời giải trên hay quá , rất ấn tượng =))

Bất đẳng thức cần chứng minh tương đương : 

$$(1-\frac{2a}{2a+b+1})+(1-\frac{2b}{2b+a+1})\geq \frac{2}{a+b+2}+\frac{1}{2}$$

$$\Leftrightarrow \frac{a+1}{2b+a+1}+\frac{b+1}{2a+b+1}\geq \frac{2}{a+b+2}+\frac{1}{2}$$

Áp dụng bất đẳng thức Cauchy-Schwarzt và Cauchy ta có : 

$$VT\geq \frac{[(a+1)+(b+1)]^2}{[(a+1)(2b+a+1)+(b+1)(2a+b+1)]}=\frac{(a+b+2)^2}{(a^2+4ab+b^2)+4(a+b)+2}\geq \frac{(a+b+2)^2}{\frac{3(a+b)^2}{2}+4(a+b)+2}$$

Đến đây đơn giản rồi , đặt t=a+b ( t > 0) thì ta quy về chứng minh :

$$\frac{(t+2)^2}{\frac{3t^2}{2}+4t+2}\geq \frac{2}{t+2}+\frac{1}{2}$$

$$\Leftrightarrow(t-2)^2(t+2)\geq 0$$

Điều này hiển nhiên đúng ,vậy bài toán được chứng minh.

Dấu bằng xảy ra khi a=b=1 .




#706955 [TOPIC] ÔN THI BẤT ĐẲNG THỨC $\boxed{\text{THPT CHUYÊN}}$...

Đã gửi bởi viet9a14124869 on 25-04-2018 - 22:05 trong Tài liệu - Đề thi

 

Bài 91:

Cho 3 số thực dương a, b, c thỏa mãn abc=1. Chứng minh:

$$ \sqrt{\frac{a}{b+3}}+\sqrt{\frac{b}{c+3}}+\sqrt{\frac{c}{a+3}} \ge \frac{3}{2} \text{   (Vasile Cirtoaje)} $$

 

Bài này mình có nghĩ đến một cách làm sau :

Trước tiên ta có một bổ đề : Với a,b,c thực không âm , ta có $3(a^2b+b^2c+c^2a) \leq (a+b+c)(a^2+b^2+c^2)$ (*)

Chứng minh : Bất đẳng thức (*) tương đương với $a(a-b)^2+b(b-c)^2+c(c-a)^2\geq 0$ ( luôn đúng ) 

Quay lại bài toán , áp dụng bất đẳng thức Cauchy-Schwarzt

LHS=$\frac{a}{\sqrt{a(b+3)}}+\frac{b}{\sqrt{b(c+3)}}+\frac{c}{\sqrt{c(a+3)}}\geq \frac{(\sqrt{a}+\sqrt{b}+\sqrt{c})^2}{\sum \sqrt{a(b+3)}}\geq \frac{(\sqrt{a}+\sqrt{b}+\sqrt{c})^2}{\sqrt{(\sum \sqrt{a}).[\sum (b+3)\sqrt{a}]}}\geq \frac{((\sqrt{a}+\sqrt{b}+\sqrt{c})^2)}{\sqrt{(\sum \sqrt{a}).[3\sum \sqrt{a}+\frac{1}{3}(\sum \sqrt{a}).\sum a]}}=\frac{(\sum \sqrt{a})}{\sqrt{3+\frac{a+b+c}{3}}}$

Do đó ta cần chứng minh :$\frac{\sum \sqrt{a}}{\sqrt{3+\frac{a+b+c}{3}}}\geq \frac{3}{2}\Leftrightarrow a+b+c+8(\sqrt{ab}+\sqrt{bc}+\sqrt{ca})\geq 27$

Bất đẳng thức cuối đúng theo bất đẳng thức Cauchy với điều kiện abc=1 .

Dấu bằng xảy ra khi a=b=c=1 .

 

P/S 01: Theo mình thì kỹ năng trình bày cũng rất quan trọng , nên cũng mong các bạn đăng bài giải có thể chia sẻ , phô diễn nó cho các bạn khác học tập theo  ^_^ ... 

 

P/S 02 : Mình nhận ra bài này cũng có thể đổi biến rồi dùng bất đẳng thức Cauchy-Schwarzt ,cũng là một cách rất ngắn gọn và đẹp =)) ....




#707237 [TOPIC] ÔN THI BẤT ĐẲNG THỨC $\boxed{\text{THPT CHUYÊN}}$...

Đã gửi bởi viet9a14124869 on 29-04-2018 - 07:43 trong Tài liệu - Đề thi

Anh viet9a14124869 lời giải bài này đâu cần phức tạp quá vậy ạ... 

Ta có:

$\frac{4}{a+b+2}+\frac{4a}{2a+b+1}+\frac{4b}{2b+a+1}=\frac{4}{(a+1)+(b+1)}+\frac{4a}{(a+b)+(a+1)}+\frac{4b}{(a+b)+b+1}\leq \frac{1}{a+1}+\frac{1}{b+1}+\frac{a}{a+b}+\frac{a}{a+1}+\frac{b}{a+b}+\frac{b}{b+1}=3$

Anh quên mất ^^ đi ngủ mới nghĩ ra cái này  :lol:  :lol: .....